LSAT and Law School Admissions Forum

Get expert LSAT preparation and law school admissions advice from PowerScore Test Preparation.

 Adam Tyson
PowerScore Staff
  • PowerScore Staff
  • Posts: 5153
  • Joined: Apr 14, 2011
|
#46996
Complete Question Explanation

The correct answer choice is (B).

See the Setup and Rules diagram thread here to see some of the key inferences of this game, namely that W is always selected last and that L will never select Y, his last choice: lsat/viewtopic.php?f=56&t=11790&p=46994#p46994

Answer choice (A): This list question is trouble because we have no information about who picks in what order, only who ends up with what. The first approach might be to use that inference about L to see if any answers can be eliminated, and sure enough, we can knock answer A out of contention for exactly that reason. The rest, not so much, so it's time to do what we do on pattern games and apply a little brute force to the answers!

Answer choice (B): This is the correct answer choice. It looks like L is the only one to get his first choice office, X, so L would go first here. T getting Y in this scenario works if he goes second, with L having gone first. If J goes third, he'll get Z since X and Y are already taken, and that leaves P to go last, getting W, as we inferred. This answer works, and it's the right answer! (As it happens, this is the exact order that I randomly tried out in the setup phase. That's just a lucky coincidence, and since you can't be counted on to have that happen for you, I refrain from suggesting an answer-by-answer approach that has you looking back at the three or four scenarios you may have come up with in that phase. Still, you might give that a try to see if any of these answers look like something you tried out).

Answer choice (C): Just to see how we might proceed if B was NOT a workable solution, let's look at answer C: L gets W, so I know he is going last. Did anyone get their first choice? Nope, nobody did, and that is enough to kill this answer and move along.

Answer choice (D): Again, L gets W so must be going last. Who got their first choice? Only J, so J must have gone first. Now if P goes second, he must pick Z, but he didn't. That means T had to go second and pick X, his first choice, since it is still available. But he didn't, so this answer won't work either.

Answer choice (E): T has W and so goes last. Who got their first choice? Only J, so he went first. Now if L went second, he would select X, but he did not. If P went second, he would select Z, but he did not. This answer is impossible, too.


B is the winner! If they had moved the answers around, making E the correct one, we would eat a lot of time going through the answer choices, but we would also likely learn a lot more about the game along the way, so it might not be the worst thing in the world. Honestly, having found B to be a working solution, this is one of those special cases where I might just pick it and move along without checking the other answers, unless I had lots and lots of time to spare. Best practice is to check all five, but when time runs short, sacrifices may sometimes be made. Just be careful!

Get the most out of your LSAT Prep Plus subscription.

Analyze and track your performance with our Testing and Analytics Package.